Symmetree

Algebra Level 3

{ a b c = 1 a + b + c = 3 1 a + 1 b + 1 c = 3 \large \begin{cases} abc = -1 \\ a+b+c=-3 \\ \dfrac{1}{a} + \dfrac{1}{b} + \dfrac{1}{c} = -3 \end{cases}

How many ordered triples ( a , b , c ) (a,b,c) of reals satisfy the system of equations above?


The answer is 1.

This section requires Javascript.
You are seeing this because something didn't load right. We suggest you, (a) try refreshing the page, (b) enabling javascript if it is disabled on your browser and, finally, (c) loading the non-javascript version of this page . We're sorry about the hassle.

3 solutions

Alex Segesta
Nov 18, 2017

Multiplying the third equation by a b c abc we get that a b + b c + c a = 3 ab+bc+ca = 3 . Then by Vieta's, a , b , c a,b,c are the roots of the equation x 3 + 3 x 2 + 3 x + 1 = 0 x^3+3x^2+3x+1 =0 which can be factored as ( x + 1 ) 3 (x+1)^3 . Hence, ( a , b , c ) = ( 1 , 1 , 1 ) (a,b,c) = (-1,-1,-1) is the only solution.

Chan Tin Ping
Nov 18, 2017

It is obvious that ( a , b , c ) = ( 1 , 1 , 1 ) (a, b, c)=(-1,-1,-1) is an answer. Let us proof that there exist this answer only.

By the third equation, we can get a b + a c + b c = 3 ab+ac+bc=3 . Now, square the first equation, we will get a 2 + b 2 + c 2 + 2 a b + 2 b c + 2 a c = 9 a 2 + b 2 + c 2 = 3 \begin{aligned} a^2+b^2+c^2+2ab+2bc+2ac&=9 \\ a^2+b^2+c^2&=3 \end{aligned}

Solution 1) Now apply Cauchy inequaliy, ( a 2 + b 2 + c 2 ) ( 1 2 + 1 2 + 1 2 ) ( a + b + c ) 2 (a^2+b^2+c^2)(1^2+1^2+1^2) \geq (a+b+c)^2 L H S = 3 × 3 = 9 LHS=3\times3=9 R H S = ( 3 ) 2 = 9 RHS=(-3)^2=9 Which means that Cauchy equality hold. Hence, a=b=c. Combine it with the first equation, we can conclude that there exist only 1 \large 1 triple, which is ( 1 , 1 , 1 ) (-1,-1,-1) .

Solution 2) We got a + b + c = 3 2 a + 2 b + 2 c = 6 a 2 + b 2 + c 2 = 3 a 2 + 2 a + b 2 + 2 b + c 2 + 2 c = 3 ( a + 1 ) 2 + ( b + 1 ) 2 + ( c + 1 ) 2 = 0 \begin{aligned} a+b+c&=-3 \\ 2a+2b+2c&=-6\\ a^2+b^2+c^2&=3 \\ a^2+2a+b^2+2b+c^2+2c&=-3\\ (a+1)^2+(b+1)^2+(c+1)^2&=0\\ \end{aligned} Hence, a + 1 = b + 1 = c + 1 = 0 a+1=b+1=c+1=0 a = b = c = 1 a=b=c=-1

Chew-Seong Cheong
Nov 19, 2017

1 a + 1 b + 1 c = 3 Given a b + b c + c a a b c = 3 Given that a b c = 1 a b + b c + c a 1 = 3 a b + b c + c a = 3 Multiply both sides by a a 2 b + a b c + c a 2 = 3 a a 2 b 1 + c a 2 = 3 a a 2 ( b + c ) = 3 a + 1 a 2 ( a + b + c a ) = 3 a + 1 Given that a + b + c = 3 a 2 ( 3 a ) = 3 a + 1 a 3 + 3 a 2 + 3 a + 1 = 0 ( a + 1 ) 3 = 0 a = 1 \begin{aligned} \frac 1a + \frac 1b + \frac 1c & = -3 & \small \color{#3D99F6} \text{Given} \\ \frac {ab+bc+ca}{\color{#3D99F6}abc} & = - 3 & \small \color{#3D99F6} \text{Given that }abc = -1 \\ \frac {ab+bc+ca}{\color{#3D99F6}-1} & = - 3 \\ ab+bc+ca & = 3 & \small \color{#3D99F6} \text{Multiply both sides by }a \\ a^2b+{\color{#3D99F6}abc} + ca^2 & = 3a \\ a^2b{\color{#3D99F6}-1} + ca^2 & = 3a \\ a^2(b+c) & = 3a+1 \\ a^2({\color{#3D99F6}a + b + c} - a) & = 3a+1 & \small \color{#3D99F6} \text{Given that }a+b+c = -3 \\ a^2({\color{#3D99F6}-3} - a) & = 3a+1 \\ a^3+3a^2+3a+1 & = 0 \\ (a+1)^3 & = 0 \\ \implies a & = -1 \end{aligned}

Since a a , b b and c c are identical in the system of equations, there is only 1 \boxed{1} triple ( 1 , 1 , 1 ) (-1,-1,-1) satisfies the system.

0 pending reports

×

Problem Loading...

Note Loading...

Set Loading...